3. hidrodinÂmica - deg.ufla.brdeg.ufla.br/site/_adm/upload/file/hidrodinamicapdf.pdf · 1 3....

16
1 3. HIDRODINÂMICA 3.1 Princípios Gerais. A Hidrodinâmica tem por objetivo geral o estudo do movimento dos fluidos. O movimento dos fluidos pode ser classificado como: ¾ Uniforme (as propriedades não variam ao longo do espaço) Acelerado ¾ Permanente (em um mesmo ponto as propriedades não variam ao longo do tempo) ¾ NãoUniforme Retardado Classificação do Movimento dos Fluidos ¾ Variado (em um mesmo ponto as propriedades variam ao longo do tempo) Em nossas aplicações, trabalharemos com movimento permanente. Velocidade: V 1 V 2 Pressão: P 1 P 2 0 dt dP dt dP dt dv dt dv 2 1 2 1 = = = = No movimento permanente as propriedades são independentes do te mpo •1 Velocidade: V 1 V 2 Pressão: P 1 P 2 0 dt dP dt dP dt dv dt dv 2 1 2 1 = = = = No movimento permanente as propriedades são independentes do te mpo •1 •2 Em uma determinada seção, a vazão ou descarga representa a quantidade de líquido que atravessa esta seção por unidade de tempo. Na hidráulica, expressamos a vazão em termos do volume de água que atravessa uma determianda seção por unidade de tempo: litros por minuto, metros cúbicos por segundo, metros cúbicos por hora etc. h m ; s m ; segundo Litros : Ex Tempo de Unidade Volume VAZAO 3 3 =

Upload: trinhduong

Post on 27-Sep-2018

235 views

Category:

Documents


0 download

TRANSCRIPT

Page 1: 3. HIDRODINÂMICA - deg.ufla.brdeg.ufla.br/site/_adm/upload/file/hidrodinamicaPDF.pdf · 1 3. HIDRODINÂMICA 3.1 Princípios Gerais. • A Hidrodinâmica tem por objetivo geral o

1

3. HIDRODINÂMICA

3.1 Princípios Gerais.

• A Hidrodinâmica tem por objetivo geral o estudo do movimento dos fluidos.

• O movimento dos fluidos pode ser classificado como:

Uniforme

(as propriedades não variam ao longo do espaço)

Acelerado

Permanente (em um mesmo ponto as propriedades não variam ao longo do

tempo) NãoUniforme Retardado

Classificação do

Movimento dos Fluidos

Variado (em um mesmo ponto as propriedades variam ao longo do tempo)

Em nossas aplicações, trabalharemos com movimento permanente. �������������������������������������������������������������

����������������������������������������������������������������������������������������������������������������������������������������������������������������������������������������������������������������������������������������������������

������������������������������������������

Velocidade: V1 V2

Pressão: P1 P2

0dt

dPdtdP

dtdv

dtdv 2121 ====

No movimento permanenteas propriedades são independentes

do tempo •1 •2

�����������������������������������������������������������������������������������������������������������������������������������������������������������������������������������������������������������������������������������������������������������������������������������������������������������������

������������������������������������������

�����������������������������������������������������������������������������������������������������������������������������������������������������������������������������������������������������������������������������������������������������������������������������������������������������������������

������������������������������������������

Velocidade: V1 V2

Pressão: P1 P2

0dt

dPdtdP

dtdv

dtdv 2121 ====

No movimento permanenteas propriedades são independentes

do tempo •1 •2

• Em uma determinada seção, a vazão ou descarga representa a quantidade de líquido que atravessa esta seção por unidade de tempo.

• Na hidráulica, expressamos a vazão em termos do volume de água

que atravessa uma determianda seção por unidade de tempo: litros por minuto, metros cúbicos por segundo, metros cúbicos por hora etc.

hm;

sm;

segundoLitros :Ex

Tempo de UnidadeVolumeVAZAO

33

=

Page 2: 3. HIDRODINÂMICA - deg.ufla.brdeg.ufla.br/site/_adm/upload/file/hidrodinamicaPDF.pdf · 1 3. HIDRODINÂMICA 3.1 Princípios Gerais. • A Hidrodinâmica tem por objetivo geral o

2

• A seguinte relação pode ser estabelecida entre a vazão, a área de escoamento e a velocidade média de escoamento da água na seção considerada.

VelocidadeAreaVazao ⋅=

sm m

sm 2

3

⋅=

VelocidadeAreaVazao ⋅=

sm m

sm 2

3

⋅=

• É importante ressaltar que a velocidade média se refere `a área da seção de escoamento, que pode ou não ser igual à área da seção tubo.

�����������������������������������������������������������������������������������������������������������������������������������������������������������������������������������������������������������������������������������������������������������������������������������������������������������������������������������������������������������������������������������������������������������������������������������������������������������������������������������������������������������������������������������������������������������������������������������������������������������������������������������������������������������������������������������������������������������������������������������������������������������������������������������������������������������������������������������������������������������������������������������������������������������������������������������������������������������������������������������������������������������������������������������������������������������������������������������������������������������������������������������������������������������������������������������������������������������������������������������������������������������������������������������������������������������������������������������������������������������������������������������������������������������������������������������������������������������������������������������������������������������������������������������������������������������������������������������������������������������������������������������������������������������������������������������������������������������������������������������������������������������������������������������������������������������������������������������������������������������������������������������������������������������������������������������������������������������������������������������������������������������������������������������������������������������������������������������������������������������������������������������������������������������������������������������������������������������������������������������������������������������������������������������������������������������������������������������������������������������������������������������������������������������������������������������������������������������������������������������������������������������������������������������������������������������������������������������������������������������������������������������������������������������������������������������������������������������������������������������������������������������������������������������������������������������������������������������������������������������������������������������������������������������������������������������������������������������������������������������������������������������������������������������������������������������������������������������������������������������������������������������������������������������������������������������������������������������������������������������������������������������������������������������������������������������������������������������������������������������������������������������������������������������������������������������������������������������������������������������������������������������������������������������������������������������������������������������������������������������������������������������������������������������������������������������������������������������������������������������������������������������������������������������������������������������������������������������������������������������������������������������������������������������������������������������������������������������������������������������������������������������������������������������������������������������������������������������������������������������������������������������������������������������������������������������������������������������������������������������������������������������������������������������������������������������������������������������������������������������������������������������������������������������������������������������������������������������������������������������������������������������������������������������������������������������������������������������������������������������������������������������������������������������������������������������������������������������������������������������������������������������������������������������������������������������������������������������������������������������������������������������������������������������������������������������������������������������������������������������������������������������������������������������������������������������������������������������������������������������������������������������������������������������������������������������������������������������������������������������������������������������������������������������������������������������������������������������������������������������������������������������������������������������������������������������������������������������������������������������������������������������������������������������������������������������������������������������������������������������������������������������������������������������������������������������������������������������������������������������������������������������������������������������������������������������������������������������������������������������������������������������������������������������������������������������������������������������������������������������������������������������������������������������������������������������������������������������������������������������������������������������������������������������������������������������������������������������������������������������������������������������������������������������������������������������������������������������������������������������������������������������������������������������������������������������������������������������������������������������������������������������������������������������������������������������������������������������������������������������������������������������������������������������������������������������������������������������������������������������������������������������������������������������������������������������������������������������������������������

�������������������������������������������������������������������������������������������������������������������������������������������������������������������������������������������������������������������������������������������������������������������������������������������������������������������������������������������������������������������������������������������������������������������������������������������������������������������������������������������������������������������������������������������������������������������������������������������������������������������������������������������������������������������������������������������������������������������������������������������������������������������������������������������������������������������������������������������������������������������������������������������������������������������������������������������������������������������������������������������������������������������������������������������������������������������������������������������������������������������������������������������������������������������������������������������������������������������������������������������������������������������������������������������������������������������������������������������������������������������������������������������������������������������������������������������������������������������������������������������������������������������������������������������������������������������������������������������������������������������������������������������������������������������������������������������������������������������������������������������������������������������������������������������������������������������������������������������������������������������������������������������������������������������������������������������������������������������������������������������������������������������������������������������������������������������������������������������������������������������������������������������������������������������������������������������������������������������������������������������������������������������������������������������������������������������������������������������������������������������������������������������������������������������������������������������������������������������������������������������������������������������������������������������������������������������������������������������������������������������������������������������������������������������������������������������������������������������������������������������������������������������������������������������������������������������������������������������������������������������������������������������������������������������������

����������������������������������������������������������������������������������������������������������������������������������������������������������������������������������������������������������������������������������������������������������������������������������������������������������������������������������������������������������������������������������������������������������������������������������������������������������������������������������������������������������������������������������������������������������������������������������������������������������������������������������������������������������������������������������������������������������������������������������������������������������������������������������������������������������������������������������������������������������������������������������������������������������������������������������������������������������������������������������������������������������������������������������������������������������������������������������������������������������������������������������������������������������������������������������������������������������������������������������������������������������������������������������������������������������������������������������������������������������������������������������������������������������������������������������������������������������������������������������������������������������������������������������������������������������������������������������������������������������������������������������������������������������������������������������������������������������������������������������������������������������������������������������������������������������������������������������������������������������������������������������������������������������������������������������������������������������������������������������������������������������������������������������������������������������������������������������������������������������������������������������������������������������������������������������������������������������������������������������������������������������������������������������������������������������������������������������������������������������������������������������������������������������������������������������������������������������������������������������������������������������������������������������������������������������������������������������������������������������������������������������������������������������������������������������������������������������������������������������������������������������������������������������������������������������������������������������������������������������������������������������������������������������������������������������������������������������������������������������������������������������������������������������������������������������������������������������������������������������������������������������������������������������������������������������������������������������������������������������������������������������������������������������������������������������������������������������������������������������������������������������������������������������������������������

�����������������������������������������������������������������������������������������������������������������������������������������������������������������������������������������������������������������������������������������������������������������������������������������������������������������������������������������������������������������������������������������������������������������������������������������������������������������������������������������������������������������������������������������������������������������������������������������������������������������������������������������������������������������������������������������������������������������������������������������������������������������������������������������������������������������������������������������������������������������������������������������������������������������������������������������������������������������������������������������������������������������������������������������������������������������������������������������������������������������������������������������������������������������������������������������������������������������������������������������������������������������������������������������������������������������������������������������������������������������������������������������������������������������������������������������������������������������������������������������������������������������������������������������������������������������������������������������������������������������������������������������������������������������������������������������������������������������������������������������������������������������������������������������������������������������������������������������������������������������������������������������������������������������������������������������������������������������������������������������������������������������������������������������������������������������������������������������������������������������������������������������������������������������������������������������������������������������������������������������������������������������������������������������������������������������������������������������������������������������������������������������������������������������������������������������������������������������������������������������������������������������������������������������������������������������������������������������������������������������������������������������������������������������������������������������������������������������������������������������������������������������������������������������������������������������������������������������������������������������������������������������������������������������������������������������������������������������������������������������������������������������������������������������������������������������������������������������������������������������������������������������������������������������������������������������������������������������������������������������������������������������������������������������������������������������������������������������������������������������������������������������������������������������������������������������������������������������������������������������������������������������������������������������������������������������������������������������������������������������������������������������������������������������������������������������������������������������������������������������������������������������������������������������������������������������������������������������������������������������������������������������������������������������������������������������������������������������������������������������������������������������������������������������������������������������������������������������������������������������������������������������������������������������������������������������������������������������������������������������������������������������������������������������������������������������������������������������������������������������������������������������������������������������������������������������������������������������������������������������������������������������������������������������������������������������������������������������������������������������������������������������������������������������������������������������������������������������������������������������������������������������������������������������������������������������������������������������������������������������������������������������������������������������������������������������������������������������������������������������������������������������������������������������������������������������������������������������������������������������������������������������������������������������������������������������������������������������������������������������������������������������������������������������������������������������������������������������������������������������������������������������������������������������������������������������������������������������������������������������������������������������������������������������������������������������������������������������������������������������������������������������������������������������������������������������������������������������������������������������������������������������������������������������������������������������������������������������������������������������������������������������������������������������������������������������������������������������������������������������������������������������������������������������������������������������������������������������������������������������������������������������������������������������������������������������������������������������������������������������������������������������������

�������������������������������������������������������������������������������������������������������������������������������������������������������������������������������������

���������������������������������������������������������������������������������������������������������������������������������������������������������������������������������������������������������������������������������������������������������������������������������������������������������������������������������������������������������������������������������������������������������������������������������������������������������������������������������������������������������������������������������������������������������������������������������������������������������������������������������������������������������������������������������������������������������������������������������������������������������������������������������������������������������������������������������������������������������������������������������������������������������������������������������������������������������������������������������������������������������������������������������������������������������������������������������������������������������������������������������������������������������������������������������������������������������������������������������������������������������������������������������������������������������������������������������������������������������������������������������������������������������������������������������������������������������������������������������������������������������������������������������������������������������������������������������������������������������������������������������������������������������������������������������������������������������������������������������������������������������������������������������������������������������������������������������������������������������������������������������������������������������������������������������������������������������������������������������������������������������������������������������������������������������������������������������������������������������������������������������������������������������������������������������������������������������������������������������������������������������������������������������������������������������������������������������������������������������������������������������������������������������������������������������������������������������������������������������������������������������������������������������������������������������������������������������������������������������������������������������������������������������������������������������������������������������������������������������������������������������������������������������������������������������������������������������������������������������������������������������������������������������������������������������������������������������������������������������������������������������������������������������������������������������������������������������������������������������������������������������������������������������������������������������������������������������������������������������������������������������������������������������������������������������������������������������������������������������������������������������������������������������������������������������������������������������������������������������������������������������������������������������������������������������������������������������������������������������������������������������������������������������������������������������������������������������������������������������������������������������������������������������������������������������������������������������������������������������������������������������������������������������������������������������������������������������������������������������������������������������������������������������������������������������������������������������������������������������������������������������������������������������������������������������������������������������������������������������������������������������������������������������������������������������������������������������������������������������������������������������������������������������������������������������������������������������������������������������������������������������������������������������������������������������������������������������������������������������������������������������������������������������������������������������������������������������������������������������������������������������������������������������������������������������������������������������������������������������������������������������������������������������������������������������������������������������������������������������������������������������������������������������������������������������������������������������������������������������������������������������������������������������������������������������������������������������������������������������������������������������������������������������������������������������������������������������������������������������������������������������������������������������������������������������������������������������������������������������������������������������������������������������������������������������������������������������������������������������������������������������������������������������������������������������������������������������������������������������������������������������������������������������������������������������������������������������������������������������������������������������������������������������������������������������������������������������������������������������������������������������������������������������������������������������������������������������������������������������������������������������������������������������������������������������������������������������������������������������������������������������������������������������������������������������������������������������������������������������������������������������������������������������������������������������������������������������������������������������������������������������������������������������������������������������������������������������������������������������������������������������������������������������������������������������������������������������������������������������������������������������������������������������������������������������������������������������������������������������������������������������������������������������������������������������������������������������������������������������������������������������������������������������������������������������������������������������������������������������������������������

�������������������������������������������������������������������������������������������������������������������������������������������������������������������������������������������������������������������������������������������������������������������������������������������������������������������������������������������������������������������������������������������������������������������������������������������������������������������������������������������������������������������������������������������������������������������������������������������������������������������������������������������������������������������������������������������������������������������������������������������������������������������������������������������������������������������������������������������������������������������������������������������������������������������������������������������������������������������������������������������������������������������������������������������������������������������������������������������������������������������������������������������������������������������������������������������������������������������������������������������������������������������������������������������������������������������������������������������������������������������������������������������������������������������������������������������������������������������������������������������������������������������������������������������������������������������������������������������������������������������������������������������������������������������������������������������������������������������������������������������������������������������������������������������������������������������������������������������������������������������������������������������������������������������������������������������������������������������������������������������������������������������������������������������������������������������������������������������������������������������������������������������������������������������������������������������������������������������������������������������������������������������������������������������������������������

������������������������������������������������������������������������������������������������������������������������������������������������������������������������������������������������������������������������������������������������������������������������������������������������������������������������������������������������������������������������������������������������������������������������������������������������������������������������������������������������������������������������������������������������������������������������������������������������������������������������������������������������������������������������������������������������������������������������������������������������������������������������������������������������������������������������������������������������������������������������������������������������������������������������������������������������������������������������������������������������������������������������������������������������������������������������������������������������������������������������������������������������������������������������������������������������������������������������������������������������������������������������������������������������������������������������������������������������������������������������������������������������������������������������������������������������������������������������������������������������������������������������������������������������������������������������������������������������������������������������������������������������������������������������������������������������������������������������������������������������������������������������������������������������������������������������������������������������������������������������������������������������������������������������������������������������������������������������������������������������������������������������������������������������������������������������������������������������������������������������������������������������������������������������������������������������������������������������������������������������������������������������������������������������������������������������������������������������������������������������������������������������������������������������������������������������������������������������������������������������������������������������������������������������������������������������������������������������������������������������������������������������������������������������������������������������������������������������������������������������������������������������������������������������������������������������������������������������������������������������������������������������������������������������������������������������������������������������������������������������������������������������������������������������������������������������������������������������������������������������������������������������������������������������������������������������������������������������������������������������������������������������������������������������������������������������������������������������������������������������������������������������������������������������������������������������������������������������������������������������������������������������������������������������������������������������������������������������������������������������������������������������������������������������������������������������������������������������������������������������������������������������������������������������������������������������������������������������������������������������������������������������������������������������������������������������������������������������������������������������������������������������������������������������������������������������������������������������������������������������������������������������������������������������������������������������������������

���������������������������������������������

=

���������������������������������������������������������������������������������������������������������������������������������������������������������������������������������������������������������������������������������������������������������������������������������������������������������������������������������������������������������������������������������������������������������������������������������������������������������������������������������������������������������������������������������������������������������������������������������������������������������������������������������������������������������������������������������������������������������������������������������������������������������������������������������������������������������������������������������������������������������������������������������������������������������������������������������������������������������������������������������������������������������������������������������������������������������������������������������������������������������������������������������������������������������������������������������������������������������������������������������������������������������������������������������������������������������������������������������������������������������������������������������������������������������������������������������������������������������������������������������������������������������������������������������������������������������������������������������������������������������������������������������������������������������������������������������������������������������������������������������������������������������������������������������������������������������������������������������������������������������������������������������������������������������������������������������������������������������������������������������������������������������������������������������������������������������������������������������������������������������������������������������������������������������������������������������������������������������������������������������������������������������������������������������������������������������������������������������������������������������������������������������������������������������������������������������������������������������������������������������������������������������������������������������������������������������������������������������������������������������������������������������������������������������������������������������������������������������������������������������������������������������������������������������������������������������������������������������������������������������������������������������������������������������������������������������������������������������������������������������������������������������������������������������������������������������������������������������������������������������������������������������������������������������������������������������������������������������������������������������������������������������������������������������������������������������������������������������������������������������������������������������������������������������������������������������������������������������������������������������������������������������������������������������������������������������������������������������������������������������������������������������������������������������������������������������������������������������������������������������������������������������������������������������������������������������������������������������������������������������������������������������������������������������������������������������������������������������������������������������������������������������������������������������������������������������������������������������������������������������������������������������������������������������������������������������������������������������������������������������������������������������������������������������������������������������������������������������������������������������������������������������������������������������������������������������������������������������������������������������������������������������������������������������������������������������������������������������������������������������������������������������������������������������������������������������������������������������������������������������������������������������������������������������������������������������������������������������������������������������������������������������������������������������������������������������������������������������������������������������������������������������������������������������������������������������������������������������������������������������������������������������������������������������������������������������������������������������������������������������������������������������������������������������������������������������������������������������������������������������������������������������������������������������������������������������������������������������������������������������������������������������������������������������������������������������������������������������������������������������������������������������������������������������������������������������������������������������������������������������������������������������������������������������������������������������������������������������������������������������������������������������������������������������������������������������������������������������������������������������������������������������������������������������������������������������������������������������������������������������������������������������������������������������������������������������������������������������������������������������������������������������������������������������������������������������������������������������������������������������������������������������������������������������������������������������������������������������������������������������������������������������������������������������������������������������������������������������������������������������������������������������������������������������������������������������������������������������������������������������������������������������������������������������������������������������������������������������������������������������������������������������������������������������������������������������������������������������������������������

����������������������������������������������������������������������������������������������������������������������������������������������������������������������������������������������������������������������������������������������������������������������������������������������������������������������������������������������������������������������������������������������������������������������������������������������������������������������������������������������������������������������������������������������������������������������������������������������������������������������������������������������������������������������������������������������������������������������������������������������������������������������������������������������������������������������������������������������������������������������������������������������������������������������������������������������������������������������������������������������������������������������������������������������������������������������������������������������������������������������������������������������������������������������������������������������������������������������������������������������������������������������������������������������������������������������������������������������������������������������������������������������������������������������������������������������������������������������������������������������������������������������������������������������������������������������������������������������������������������������������������������������������������������������������������������������������������������������������������������������������������������������������������������������������������������������������������������������������������������������������������������������������������������������������������������������������������������������������������������������������������������������������������������������������������������������������������������������������������������������������������������������������������������������������������������������������������������������������������������������������������������������������������������������������������������������������������������������������������������������������

������������������������������������������������������������������������������������������������������������������������������������������������������������������������������������������������������������������������������������������������������������������������������������������������������������������������������������������������������������������������������������������������������������������������������������������������������������������������������������������������������������������������������������������������������������������������������������������������������������������������������������������������������������������������������������������������������������������������������������������������������������������������������������������������������������������������������������������������������������������������������������������������������������������������������������������������������������������������������������������������������������������������������������������������������������������������������������������������������������������������������������������������������������������������������������������������������������������������������������������������������������������������������������������������������������������������������������������������������������������������������������������������������������������������������������������������������������������������������������������������������������������������������������������������������������������������������������������������������������������������������������������������������������������������������������������������������������������������������������������������������������������������������������������������������������������������������������������������������������������������������������������������������������������������������������������������������������������������������������������������������������������������������������������������������������������������������������������������������������������������������������������������������������������������������������������������������������������������������������������������������������������������������������������������������������������������������������������������������������������������������������������������������������������������������������������������������������������������������������������������������������������������������������������������������������������������������������������������������������������������������������������������������������������������������������������������������������������������������������������������������������������������������������������������������������������������������������������������������������������������������������������������������������������������������������������������������������������������������������������������������������������������������������������������������������������������������������������������������������������������������������������������������������������������������������������������������������������������������������������������������������������������������������������������������������������������������������������������������������������������������������������������������������������������������������������������������������������������������������������������������������������������������������������������������������������������������������������������������������������������������������������������������������������������������������������������������������������������������������������������������������������������������������������������������������������������������������������������������������������������������������������������������������������������������������������������������������������������������������������������������������������������������������������������������������������������������������������������������������������������������������������������������������������������������

Área da Seção de

Escoamento ������������������������������������������������������������

���������������������������������������������������������������������������������������������������������������������������������������������������������������������������������������������������������������������������������������������������������������������������������������������������������������������������������������������������������������������������������������������������������������������������������������������������������������������������������������������������������������������������������������������������������������������������������������������������������������������������������������������������������������������������������������������������������������������������������������������������������������������������������������������������������������������������������������������������������������������������������������������������������������������������������������������������������������������������������������������������������������������������������������������������������������������������������������������������������������������������������������������������������������������������������������������������������������������������������������������������������������������������������������������������������������������������������������������������������������������������������������������������������������������������������������������������������������������������������������������������������������������������������������������������������������������������������������������������������������������������������������������������������������������������������������������������������������������������������������������������������������������������������������������������������������������������������������������������������������������������������������������������������������������������������������������������������������������������������������������������������������������������������������������������������������������������������������������������������������������������������������������������������������������������������������������������������������������������������������������������������������������������������������������������������������������������������������������������������������������������������������������������������������������������������������������������������������������������������������������������������������������������������������������������������������������������������������������������������������������������������������������������������������������������������������������������������������������������������������������������������������������������������������������������������������������������������������������������������������������������������������������������������������������������������������������������������������������������������������������������������������������������������������������������������������������������������������������������������������������������������������������������������������������������������������������������������������������������������������������������������������������������������������������������������������������������������������������������������������������������������������������������������������������������������������������������������������������������������������������������������������������������������������������������������������������������������������������������������������������������������������������������������������������������������������������������������������������������������������������������������������������������������������������������������������������������������������������������������������������������������������������������������������������������������������������������������������������������������������������������������������������������������������������������������������������������������������������������������������������������������������������������������������������������������������������������������������������������������������������������������������������������������������������������������������������������������������������������������������������������������������������������������������������������������������������������������������������������������������������������������������������������������������������������������������������������������������������������������������������������������������������������������������������������������������������������������������������������������������������������������������������������������������������������������������������������������������������������������������������������������������������������������������������������������������������������������������������������������������������������������������������������������������������������������������������������������������������������������������������������������������������������������������������������������������������������������������������������������������������������������������������������������������������������������������������������������������������������������������������������������������������������������������������������������������������������������������������������������������������������������������������������������������������������������������������������������������������������������������������������������������������������������������������������������������������������������������������������������������������������������������������������������������������������������������������������������������������������������������������������������������������������������������������������������������������������������������������������������������������������������������������������������������������������������������������������������������������������������������������������������������������������������������������������������������������������������������������������������������������������������������������������������������������������������������������������������������������������������������������������������������������������������������������������������������������������������������������������������������������������������������������������������������������������������������������������������������������������������������������������������������������������������������������������������������������������������������������������������������������������������������������������������������������������������������������������������������������������������������������������������������������������������������������������������������������������������������������������������������������������������������������������������������������������������������������������������������������������������������������������������������������������������������������������������������������������������������������������������������������������������������������������������������������������������������������������������������������������������

��������������������������������������������������������������������������������������������������������������������������������������������������������������������������������������������������������������������������������������������������������������������������������������������������������������������������������������������������������������������������������������������������������������������������������������������������������������������������������������������������������������������������������������������������������������������������������������������������������������������������������������������������������������������������������������������������������������������������������������������������������������������������������������������������������������������������������������������������������������������������������������������������������������������������������������������������������������������������������������������������������������������������������������������������������������������������������������������������������������������������������������������������������������������������������������������������������������������������������������������������������������������������������������������������������������������������������������������������������������������������������������������������������������������������������������������������������������������������������������������������������������������������������������������������������������������������������������������������������������������������������������������������������������������������������������������������������������������������������������������������������������������������������������������������������������������������������������������������������������������������������������������������������������������������������������������������������������������������������������������������������������������������������������������������������������������������������������������������������������������������������������������������������������������������������������������������������������������������������������������������������������������������������������������������������������������������������������������������������������������������������������������������������������������������������������������������������������������������������������������������������������������������������������������������������������������������������������������������������������������������������������������������������������������������������������������������������������������������������������������������������������������������������������������������������������������������������������������������������������������������������������������������������������������������������������������������������������������������������������������������������������������������������������������������������������������������������������������������������������������������������������������������������������������������������������������������������������������������������������������������������������������������������������������������������������������������������������������������������������������������������������������������������������������������������������������������������������������������������������������������������������������������������������������������������������������������������������������������������������������������������������������������������������������������������������������������������������������������������������������������������������������������������������������������������������������������������������������������������������������������������������������������������������������������������������������������������������������������������������������������������������������������������������������������������������������������������������������������������������������������������������������������������������������������������������������������������������������������������������������������������������������������������������������������������������������������������������������������������������������������������������������������������������������������������������������������������������������������������������������������������������������������������������������������������������������������������������������������������������������������������������������������������������������������������������������������������������������������������������������������������������������������������������������������������������������������������������������������������������������������������������������������������������������������������������������������������������������������������������������������������������������������������������������������������������������������������������������������������������������������������������������������������������������������������������������������������������������������������������������������������������������������������������������������������������������������������������������������������������������������������������������������������������������������������������������������������������������������������������������������������������������������������������������������������������������������������������������������������������������������������������������������������������������������������������������������������������������������������������������������������������������������������������������������������������������������������������������������������������������������������������������������������������������������������������������������������������������������������������������������������������������������������������������������������������������������������������������������������������������������������������������������������

��������������������������������������������������������������������������������������������������������������������������������������������������������������������������������������������������������������������������������������������������������������������������������������������������������������������������������������������������������������

< Área da Seção do

Tubo

Área da Seção de

Escoamento

Área da Seção do

Tubo

�����������������������������������������������������������������������������������������������������������������������������������������������������������������������������������������������������������������������������������������������������������������������������������������������������������������������������������������������������������������������������������������������������������������������������������������������������������������������������������������������������������������������������������������������������������������������������������������������������������������������������������������������������������������������������������������������������������������������������������������������������������������������������������������������������������������������������������������������������������������������������������������������������������������������������������������������������������������������������������������������������������������������������������������������������������������������������������������������������������������������������������������������������������������������������������������������������������������������������������������������������������������������������������������������������������������������������������������������������������������������������������������������������������������������������������������������������������������������������������������������������������������������������������������������������������������������������������������������������������������������������������������������������������������������������������������������������������������������������������������������������������������������������������������������������������������������������������������������������������������������������������������������������������������������������������������������������������������������������������������������������������������������������������������������������������������������������������������������������������������������������������������������������������������������������������������������������������������������������������������������������������������������������������������������������������������������������������������������������������������������������������������������������������������������������������������������������������������������������������������������������������������������������������������������������������������������������������������������������������������������������������������������������������������������������������������������������������������������������������������������������������������������������������������������������������������������������������������������������������������������������������������������������������������������������������������������������������������������������������������������������������������������������������������������������������������������������������������������������������������������������������������������������������������������������������������������������������������������������������������������������������������������������������������������������������������������������������������������������������������������������������������������������������������������������������������������������������������������������������������������������������������������������������������������������������������������������������������������������������������������������������������������������������������������������������������������������������������������������������������������������������������������������������������������������������������������������������������������������������������������������������������������������������������������������������������������������������������������������������������������������������������������������������������������������������������������������������������������������������������������������������������������������������������������������������������������������������������������������������������������������������������������������������������������������������������������������������������������������������������������������������������������������������������������������������������������������������������������������������������������������������������������������������������������������������������������������������������������������������������������������������������������������������������������������������������������������������������������������������������������������������������������������������������������������������������������������������������������������������������������������������������������������������������������������������������������������������������������������������������������������������������������������������������������������������������������������������������������������������������������������������������������������������������������������������������������������������������������������������������������������������������������������������������������������������������������������������������������������������������������������������������������������������������������������������������������������������������������������������������������������������������������������������������������������������������������������������������������������������������������������������������������������������������������������������������������������������������������������������������������������������������������������������������������������������������������������������������������������������������������������������������������������������������������������������������������������������������������������������������������������������������������������������������������������������������������������������������������������������������������������������������������������������������������������������������������������������������������������������������������������������������������������������������������������������������������������������������������������������������������������������������������������������������������������������������������������������������������������������������������������������������������������������������������������������������������������������������������������������������������������������������������������������������������������������������������������������������������������������������������������������������������������������������������������������������������������������������������������

�������������������������������������������������������������������������������������������������������������������������������������������������������������������������������������������������������������������������������������������������������������������������������������������������������������������������������������������������������������������������������������������������������������������������������������������������������������������������������������������������������������������������������������������������������������������������������������������������������������������������������������������������������������������������������������������������������������������������������������������������������������������������������������������������������������������������������������������������������������������������������������������������������������������������������������������������������������������������������������������������������������������������������������������������������������������������������������������������������������������������������������������������������������������������������������������������������������������������������������������������������������������������������������������������������������������������������������������������������������������������������������������������������������������������������������������������������������������������������������������������������������������������������������������������������������������������������������������������������������������������������������������������������������������������������������������������������������������������������������������������������������������������������������������������������������������������������������������������������������������������������������������������������������������������������������������������������������������������������������������������������������������������������������������������������������������������������������������������������������������������������������������������������������������������������������������������������������������������������������������������������������������������������������������������������������������������������������������������������������������������������������������������������������������������������������������������������������������������������������������������������������������������������������������������������������������������������������������������������������������������������������������������������������������������������������������������������������������������������������������������������������������������������������������������������������������������������������������������������������������������������������������������������������������������

����������������������������������������������������������������������������������������������������������������������������������������������������������������������������������������������������������������������������������������������������������������������������������������������������������������������������������������������������������������������������������������������������������������������������������������������������������������������������������������������������������������������������������������������������������������������������������������������������������������������������������������������������������������������������������������������������������������������������������������������������������������������������������������������������������������������������������������������������������������������������������������������������������������������������������������������������������������������������������������������������������������������������������������������������������������������������������������������������������������������������������������������������������������������������������������������������������������������������������������������������������������������������������������������������������������������������������������������������������������������������������������������������������������������������������������������������������������������������������������������������������������������������������������������������������������������������������������������������������������������������������������������������������������������������������������������������������������������������������������������������������������������������������������������������������������������������������������������������������������������������������������������������������������������������������������������������������������������������������������������������������������������������������������������������������������������������������������������������������������������������������������������������������������������������������������������������������������������������������������������������������������������������������������������������������������������������������������������������������������������������������������������������������������������������������������������������������������������������������������������������������������������������������������������������������������������������������������������������������������������������������������������������������������������������������������������������������������������������������������������������������������������������������������������������������������������������������������������������������������������������������������������������������������������������������������������������������������������������������������������������������������������������������������������������������������������������������������������������������������������������������������������������������������������������������������������������������������������������������������������������������������������������������������������������������������������������������������������������������������������������������������������������������������������������

�����������������������������������������������������������������������������������������������������������������������������������������������������������������������������������������������������������������������������������������������������������������������������������������������������������������������������������������������������������������������������������������������������������������������������������������������������������������������������������������������������������������������������������������������������������������������������������������������������������������������������������������������������������������������������������������������������������������������������������������������������������������������������������������������������������������������������������������������������������������������������������������������������������������������������������������������������������������������������������������������������������������������������������������������������������������������������������������������������������������������������������������������������������������������������������������������������������������������������������������������������������������������������������������������������������������������������������������������������������������������������������������������������������������������������������������������������������������������������������������������������������������������������������������������������������������������������������������������������������������������������������������������������������������������������������������������������������������������������������������������������������������������������������������������������������������������������������������������������������������������������������������������������������������������������������������������������������������������������������������������������������������������������������������������������������������������������������������������������������������������������������������������������������������������������������������������������������������������������������������������������������������������������������������������������������������������������������������������������������������������������������������������������������������������������������������������������������������������������������������������������������������������������������������������������������������������������������������������������������������������������������������������������������������������������������������������������������������������������������������������������������������������������������������������������������������������������������������������������������������������������������������������������������������������������������������������������������������������������������������������������������������������������������������������������������������������������������������������������������������������������������������������������������������������������������������������������������������������������������������������������������������������������������������������������������������������������������������������������������������������������������������������������������������������������������������������������������������������������������������������������������������������������������������������������������������������������������������������������������������������������������������������������������������������������������������������������������������������������������������������������������������������������������������������������������������������������������������������������������������������������������������������������������������������������������������������������������������������������������������������������������������������������������������������������������������������������������������������������������������������������������������������������������������������������������������������������������������������������������������������������������������������������������������������������������������������������������������������������������������������������������������������������������������������������������������������������������������������������������������������������������������������������������������������������������������������������������������������������������������������������������������������������������������������������������������������������������������������������������������������������������������������������������������������������������������������������������������������������������������������������������������������������������������������������������������������������������������������������������������������������������������������������������������������������������������������������������������������������������������������������������������������������������������������������������������������������������������������������������������������������������������������������������������������������������������������������������������������������������������������������������������������������������������������������������������������������������������������������������������������������������������������������������������������������������������������������������������������������������������������������������������������������������������������������������������������������������������������������������������������������������������������������������������������������������������������������������������������������������������������������������������������������������������������������������������������������������������������������������������������������������������������������������������������������������������������������������������������������������������������������������������������������������������������������������������������������������������������������������������������������������������������

�������������������������������������������������������������������������������������������������������������������������������������������������������������������������������������

�������������������������������������������������������������������������������������������������������������������������������������������������������������������������������������������������������������������������������������������������������������������������������������������������������������������������������������������������������������������������������������������������������������������������������������������������������������������������������������������������������������������������������������������������������������������������������������������������������������������������������������������������������������������������������������������������������������������������������������������������������������������������������������������������������������������������������������������������������������������������������������������������������������������������������������������������������������������������������������������������������������������������������������������������������������������������������������������������������������������������������������������������������������������������������������������������������������������������������������������������������������������������������������������������������������������������������������������������������������������������������������������������������������������������������������������������������������������������������������������������������������������������������������������������������������������������������������������������������������������������������������������������������������������������������������������������������������������������������������������������������������������������������������������������������������������������������������������������������������������������������������������������������������������������������������������������������������������������������������������������������������������������������������������������������������������������������������������������������������������������������������������������������������������������������������������������������������������������������������������������������������������������������������������������������������������������������������������������������������������������������������������������������������������������������������������������������������������������������������������������������������������������������������������������������������������������������������������������������������������������������������������������������������������������������������������������������������������������������������������������������������������������������������������������������������������������������������������������������������������������������������������������������������������������

����������������������������������������������������������������������������������������������������������������������������������������������������������������������������������������������������������������������������������������������������������������������������������������������������������������������������������������������������������������������������������������������������������������������������������������������������������������������������������������������������������������������������������������������������������������������������������������������������������������������������������������������������������������������������������������������������������������������������������������������������������������������������������������������������������������������������������������������������������������������������������������������������������������������������������������������������������������������������������������������������������������������������������������������������������������������������������������������������������������������������������������������������������������������������������������������������������������������������������������������������������������������������������������������������������������������������������������������������������������������������������������������������������������������������������������������������������������������������������������������������������������������������������������������������������������������������������������������������������������������������������������������������������������������������������������������������������������������������������������������������������������������������������������������������������������������������������������������������������������������������������������������������������������������������������������������������������������������������������������������������������������������������������������������������������������������������������������������������������������������������������������������������������������������������������������������������������������������������������������������������������������������������������������������������������������������������������������������������������������������������������������������������������������������������������������������������������������������������������������������������������������������������������������������������������������������������������������������������������������������������������������������������������������������������������������������������������������������������������������������������������������������������������������������������������������������������������������������������������������������������������������������������������������������������������������������������������������������������������������������������������������������������������������������������������������������������������������������������������������������������������������������������������������������������������������������������������������������������������������������������������������������������������������������������������������������������������������������������������������������������������������������������������������������������

�����������������������������������������������������������������������������������������������������������������������������������������������������������������������������������������������������������������������������������������������������������������������������������������������������������������������������������������������������������������������������������������������������������������������������������������������������������������������������������������������������������������������������������������������������������������������������������������������������������������������������������������������������������������������������������������������������������������������������������������������������������������������������������������������������������������������������������������������������������������������������������������������������������������������������������������������������������������������������������������������������������������������������������������������������������������������������������������������������������������������������������������������������������������������������������������������������������������������������������������������������������������������������������������������������������������������������������������������������������������������������������������������������������������������������������������������������������������������������������������������������������������������������������������������������������������������������������������������������������������������������������������������������������������������������������������������������������������������������������������������������������������������������������������������������������������������������������������������������������������������������������������������������������������������������������������������������������������������������������������������������������������������������������������������������������������������������������������������������������������������������������������������������������������������������������������������������������������������������������������������������������������������������������������������������������������������������������������������������������������������������������������������������������������������������������������������������������������������������������������������������������������������������������������������������������������������������������������������������������������������������������������������������������������������������������������������������������������������������������������������������������������������������������������������������������������������������������������������������������������������������������������������������������������������������������������������������������������������������������������������������������������������������������������������������������������������������������������������������������������������������������������������������������������������������������������������������������������������������������������������������������������������������������������������������������������������������������������������������������������������������������������������������������������������������������������������������������������������������������������������������������������������������������������������������������������������������������������������������������������������������������������������������������������������������������������������������������������������������������������������������������������������������������������������������������������������������������������������������������������������������������������������������������������������������������������������������������������������������������������������������������������������������������������������������������������������������������������������������������������������������������������������������������������������������������������������������������������������������������������������������������������������������������������������������������������������������������������������������������������������������������������������������������������������������������������������������������������������������������������������������������������������������������������������������������������������������������������������������������������������������������������������������������������������������������������������������������������������������������������������������������������������������������������������������������������������������������������������������������������������������������������������������������������������������������������������������������������������������������������������������������������������������������������������������������������������������������������������������������������������������������������������������������������������������������������������������������������������������������������������������������������������������������������������������������������������������������������������������������������������������������������������������������������������������������������������������������������������������������������������������������������������������������������������������������������������������������������������������������������������������������������������������������������������������������������������������������������������������������������������������������������������������������������������������������������������������������������������������������������������������������������������������������������������������������������������������������������������������������������������������������������������������������������������������������������������������������������������������������������������������������������������������������������������������������������������������������������������������������������������������������������������

�������������������������������������������������������������������������������������������������������������������������������������������������������������������������������������

���������������������������������������������������������������������������������������������������������������������������������������������������������������������������������������������������������������������������������������������������������������������������������������������������������������������������������������������������������������������������������������������������������������������������������������������������������������������������������������������������������������������������������������������������������������������������������������������������������������������������������������������������������������������������������������������������������������������������������������������������������������������������������������������������������������������������������������������������������������������������������������������������������������������������������������������������������������������������������������������������������������������������������������������������������������������������������������������������������������������������������������������������������������������������������������������������������������������������������������������������������������������������������������������������������������������������������������������������������������������������������������������������������������������������������������������������������������������������������������������������������������������������������������������������������������������������������������������������������������������������������������������������������������������������������������������������������������������������������������������������������������������������������������������������������������������������������������������������������������������������������������������������������������������������������������������������������������������������������������������������������������������������������������������������������������������������������������������������������������������������������������������������������������������������������������������������������������������������������������������������������������������������������������������������������������������������������������������������������������������������������������������������������������������������������������������������������������������������������������������������������������������������������������������������������������������������������������������������������������������������������������������������������������������������������������������������������������������������������������������������������������������������������������������������������������������������������������������������������������������������������������������������������������������������������������������������������������������������������������������������������������������������������������������������������������������������������������������������������������������������������������������������������������������������������������������������������������������������������������������������������������������������������������������������������������������������������������������������������������������������������������������������������������������������������������������������������������������������������������������������������������������������������������������������������������������������������������������������������������������������������������������������������������������������������������������������������������������������������������������������������������������������������������������������������������������������������������������������������������������������������������������������������������������������������������������������������������������������������������������������������������������������������������������������������������������������������������������������������������������������������������������������������������������������������������������������������������������������������������������������������������������������������������������������������������������������������������������������������������������������������������������������������������������������������������������������������������������������������������������������������������������������������������������������������������������������������������������������������������������������������������������������������������������������������������������������������������������������������������������������������������������������������������������������������������������������������������������������������������������������������������������������������������������������������������������������������������������������������������������������������������������������������������������������������������������������������������������������������������������������������������������������������������������������������������������������������������������������������������������������������������������������������������������������������������������������������������������������������������������������������������������������������������������������������������������������������������������������������������������������������������������������������������������������������������������������������������������������������������������������������������������������������������������������������������������������������������������������������������������������������������������������������������������������������������������������������������������������������������������������������������������������������������������������������������������������������������������������������������������������������������������������������������������������������������������������������������������������������������������������������������������������������������������������������������������������������������������������������������������������������������������������������������������������������������������������������������������������������������������������������������������������������������������������������������������������������������������������������������������������������������������������������������������������������������������������������������������������������������������������������������������������������������������������������������������������������������������������������������������������������������������������������������������������������������������������������������������������������������������������������������������������������������������������������������������������������������������������������������������������������������������������������������������������������������������������������������������������������������������������

�������������������������������������������������������������������������������������������������������������������������������������������������������������������������������������������������������������������������������������������������������������������������������������������������������������������������������������������������������������������������������������������������������������������������������������������������������������������������������������������������������������������������������������������������������������������������������������������������������������������������������������������������������������������������������������������������������������������������������������������������������������������������������������������������������������������������������������������������������������������������������������������������������������������������������������������������������������������������������������������������������������������������������������������������������������������������������������������������������������������������������������������������������������������������������������������������������������������������������������������������������������������������������������������������������������������������������������������������������������������������������������������������������������������������������������������������������������������������������������������������������������������������������������������������������������������������������������������������������������������������������������������������������������������������������������������������������������������������������������������������������������������������������������������������������������������������������������������������������������������������������������������������������������������������������������������������������������������������������������������������������������������������������������������������������������������������������������������������������������������������������������������������������������������������������������������������������������������������������������������������������������������������������������������������������������

������������������������������������������������������������������������������������������������������������������������������������������������������������������������������������������������������������������������������������������������������������������������������������������������������������������������������������������������������������������������������������������������������������������������������������������������������������������������������������������������������������������������������������������������������������������������������������������������������������������������������������������������������������������������������������������������������������������������������������������������������������������������������������������������������������������������������������������������������������������������������������������������������������������������������������������������������������������������������������������������������������������������������������������������������������������������������������������������������������������������������������������������������������������������������������������������������������������������������������������������������������������������������������������������������������������������������������������������������������������������������������������������������������������������������������������������������������������������������������������������������������������������������������������������������������������������������������������������������������������������������������������������������������������������������������������������������������������������������������������������������������������������������������������������������������������������������������������������������������������������������������������������������������������������������������������������������������������������������������������������������������������������������������������������������������������������������������������������������������������������������������������������������������������������������������������������������������������������������������������������������������������������������������������������������������������������������������������������������������������������������������������������������������������������������������������������������������������������������������������������������������������������������������������������������������������������������������������������������������������������������������������������������������������������������������������������������������������������������������������������������������������������������������������������������������������������������������������������������������������������������������������������������������������������������������������������������������������������������������������������������������������������������������������������������������������������������������������������������������������������������������������������������������������������������������������������������������������������������������������������������������������������������������������������������������������������������������������������������������������������������������������������������������������������������������������������������������������������������������������������������������������������������������������������������������������������������������������������������������������������������������������������������������������������������������������������������������������������������������������������������������������������������������������������������������������������������������������������������������������������������������������������������������������������������������������������������������������������������������������������������������������������������������������������������������������������������������������������������������������������������������������������������������������������������������������������������������

���������������������������������������������

=

���������������������������������������������������������������������������������������������������������������������������������������������������������������������������������������������������������������������������������������������������������������������������������������������������������������������������������������������������������������������������������������������������������������������������������������������������������������������������������������������������������������������������������������������������������������������������������������������������������������������������������������������������������������������������������������������������������������������������������������������������������������������������������������������������������������������������������������������������������������������������������������������������������������������������������������������������������������������������������������������������������������������������������������������������������������������������������������������������������������������������������������������������������������������������������������������������������������������������������������������������������������������������������������������������������������������������������������������������������������������������������������������������������������������������������������������������������������������������������������������������������������������������������������������������������������������������������������������������������������������������������������������������������������������������������������������������������������������������������������������������������������������������������������������������������������������������������������������������������������������������������������������������������������������������������������������������������������������������������������������������������������������������������������������������������������������������������������������������������������������������������������������������������������������������������������������������������������������������������������������������������������������������������������������������������������������������������������������������������������������������������������������������������������������������������������������������������������������������������������������������������������������������������������������������������������������������������������������������������������������������������������������������������������������������������������������������������������������������������������������������������������������������������������������������������������������������������������������������������������������������������������������������������������������������������������������������������������������������������������������������������������������������������������������������������������������������������������������������������������������������������������������������������������������������������������������������������������������������������������������������������������������������������������������������������������������������������������������������������������������������������������������������������������������������������������������������������������������������������������������������������������������������������������������������������������������������������������������������������������������������������������������������������������������������������������������������������������������������������������������������������������������������������������������������������������������������������������������������������������������������������������������������������������������������������������������������������������������������������������������������������������������������������������������������������������������������������������������������������������������������������������������������������������������������������������������������������������������������������������������������������������������������������������������������������������������������������������������������������������������������������������������������������������������������������������������������������������������������������������������������������������������������������������������������������������������������������������������������������������������������������������������������������������������������������������������������������������������������������������������������������������������������������������������������������������������������������������������������������������������������������������������������������������������������������������������������������������������������������������������������������������������������������������������������������������������������������������������������������������������������������������������������������������������������������������������������������������������������������������������������������������������������������������������������������������������������������������������������������������������������������������������������������������������������������������������������������������������������������������������������������������������������������������������������������������������������������������������������������������������������������������������������������������������������������������������������������������������������������������������������������������������������������������������������������������������������������������������������������������������������������������������������������������������������������������������������������������������������������������������������������������������������������������������������������������������������������������������������������������������������������������������������������������������������������������������������������������������������������������������������������������������������������������������������������������������������������������������������������������������������������������������������������������������������������������������������������������������������������������������������������������������������������������������������������������������������������������������������������������������������������������������������������������������������������������������������������������������������������������������������������������������������������������������������������������������������������������������������������������������������������������������������������������������������������������������������������������������������

����������������������������������������������������������������������������������������������������������������������������������������������������������������������������������������������������������������������������������������������������������������������������������������������������������������������������������������������������������������������������������������������������������������������������������������������������������������������������������������������������������������������������������������������������������������������������������������������������������������������������������������������������������������������������������������������������������������������������������������������������������������������������������������������������������������������������������������������������������������������������������������������������������������������������������������������������������������������������������������������������������������������������������������������������������������������������������������������������������������������������������������������������������������������������������������������������������������������������������������������������������������������������������������������������������������������������������������������������������������������������������������������������������������������������������������������������������������������������������������������������������������������������������������������������������������������������������������������������������������������������������������������������������������������������������������������������������������������������������������������������������������������������������������������������������������������������������������������������������������������������������������������������������������������������������������������������������������������������������������������������������������������������������������������������������������������������������������������������������������������������������������������������������������������������������������������������������������������������������������������������������������������������������������������������������������������������������������������������������������������������

������������������������������������������������������������������������������������������������������������������������������������������������������������������������������������������������������������������������������������������������������������������������������������������������������������������������������������������������������������������������������������������������������������������������������������������������������������������������������������������������������������������������������������������������������������������������������������������������������������������������������������������������������������������������������������������������������������������������������������������������������������������������������������������������������������������������������������������������������������������������������������������������������������������������������������������������������������������������������������������������������������������������������������������������������������������������������������������������������������������������������������������������������������������������������������������������������������������������������������������������������������������������������������������������������������������������������������������������������������������������������������������������������������������������������������������������������������������������������������������������������������������������������������������������������������������������������������������������������������������������������������������������������������������������������������������������������������������������������������������������������������������������������������������������������������������������������������������������������������������������������������������������������������������������������������������������������������������������������������������������������������������������������������������������������������������������������������������������������������������������������������������������������������������������������������������������������������������������������������������������������������������������������������������������������������������������������������������������������������������������������������������������������������������������������������������������������������������������������������������������������������������������������������������������������������������������������������������������������������������������������������������������������������������������������������������������������������������������������������������������������������������������������������������������������������������������������������������������������������������������������������������������������������������������������������������������������������������������������������������������������������������������������������������������������������������������������������������������������������������������������������������������������������������������������������������������������������������������������������������������������������������������������������������������������������������������������������������������������������������������������������������������������������������������������������������������������������������������������������������������������������������������������������������������������������������������������������������������������������������������������������������������������������������������������������������������������������������������������������������������������������������������������������������������������������������������������������������������������������������������������������������������������������������������������������������������������������������������������������������������������������������������������������������������������������������������������������������������������������������������������������������������������������������������������

Área da Seção de

Escoamento ������������������������������������������������������������

���������������������������������������������������������������������������������������������������������������������������������������������������������������������������������������������������������������������������������������������������������������������������������������������������������������������������������������������������������������������������������������������������������������������������������������������������������������������������������������������������������������������������������������������������������������������������������������������������������������������������������������������������������������������������������������������������������������������������������������������������������������������������������������������������������������������������������������������������������������������������������������������������������������������������������������������������������������������������������������������������������������������������������������������������������������������������������������������������������������������������������������������������������������������������������������������������������������������������������������������������������������������������������������������������������������������������������������������������������������������������������������������������������������������������������������������������������������������������������������������������������������������������������������������������������������������������������������������������������������������������������������������������������������������������������������������������������������������������������������������������������������������������������������������������������������������������������������������������������������������������������������������������������������������������������������������������������������������������������������������������������������������������������������������������������������������������������������������������������������������������������������������������������������������������������������������������������������������������������������������������������������������������������������������������������������������������������������������������������������������������������������������������������������������������������������������������������������������������������������������������������������������������������������������������������������������������������������������������������������������������������������������������������������������������������������������������������������������������������������������������������������������������������������������������������������������������������������������������������������������������������������������������������������������������������������������������������������������������������������������������������������������������������������������������������������������������������������������������������������������������������������������������������������������������������������������������������������������������������������������������������������������������������������������������������������������������������������������������������������������������������������������������������������������������������������������������������������������������������������������������������������������������������������������������������������������������������������������������������������������������������������������������������������������������������������������������������������������������������������������������������������������������������������������������������������������������������������������������������������������������������������������������������������������������������������������������������������������������������������������������������������������������������������������������������������������������������������������������������������������������������������������������������������������������������������������������������������������������������������������������������������������������������������������������������������������������������������������������������������������������������������������������������������������������������������������������������������������������������������������������������������������������������������������������������������������������������������������������������������������������������������������������������������������������������������������������������������������������������������������������������������������������������������������������������������������������������������������������������������������������������������������������������������������������������������������������������������������������������������������������������������������������������������������������������������������������������������������������������������������������������������������������������������������������������������������������������������������������������������������������������������������������������������������������������������������������������������������������������������������������������������������������������������������������������������������������������������������������������������������������������������������������������������������������������������������������������������������������������������������������������������������������������������������������������������������������������������������������������������������������������������������������������������������������������������������������������������������������������������������������������������������������������������������������������������������������������������������������������������������������������������������������������������������������������������������������������������������������������������������������������������������������������������������������������������������������������������������������������������������������������������������������������������������������������������������������������������������������������������������������������������������������������������������������������������������������������������������������������������������������������������������������������������������������������������������������������������������������������������������������������������������������������������������������������������������������������������������������������������������������������������������������������������������������������������������������������������������������������������������������������������������������������������������������������������������������������������������������������������������������������������������������������������������������������������������������������������������������������������������������������������������������������������������������������������������������������������������������������������������������������������������������������������������������������������������������������������������������������

��������������������������������������������������������������������������������������������������������������������������������������������������������������������������������������������������������������������������������������������������������������������������������������������������������������������������������������������������������������������������������������������������������������������������������������������������������������������������������������������������������������������������������������������������������������������������������������������������������������������������������������������������������������������������������������������������������������������������������������������������������������������������������������������������������������������������������������������������������������������������������������������������������������������������������������������������������������������������������������������������������������������������������������������������������������������������������������������������������������������������������������������������������������������������������������������������������������������������������������������������������������������������������������������������������������������������������������������������������������������������������������������������������������������������������������������������������������������������������������������������������������������������������������������������������������������������������������������������������������������������������������������������������������������������������������������������������������������������������������������������������������������������������������������������������������������������������������������������������������������������������������������������������������������������������������������������������������������������������������������������������������������������������������������������������������������������������������������������������������������������������������������������������������������������������������������������������������������������������������������������������������������������������������������������������������������������������������������������������������������������������������������������������������������������������������������������������������������������������������������������������������������������������������������������������������������������������������������������������������������������������������������������������������������������������������������������������������������������������������������������������������������������������������������������������������������������������������������������������������������������������������������������������������������������������������������������������������������������������������������������������������������������������������������������������������������������������������������������������������������������������������������������������������������������������������������������������������������������������������������������������������������������������������������������������������������������������������������������������������������������������������������������������������������������������������������������������������������������������������������������������������������������������������������������������������������������������������������������������������������������������������������������������������������������������������������������������������������������������������������������������������������������������������������������������������������������������������������������������������������������������������������������������������������������������������������������������������������������������������������������������������������������������������������������������������������������������������������������������������������������������������������������������������������������������������������������������������������������������������������������������������������������������������������������������������������������������������������������������������������������������������������������������������������������������������������������������������������������������������������������������������������������������������������������������������������������������������������������������������������������������������������������������������������������������������������������������������������������������������������������������������������������������������������������������������������������������������������������������������������������������������������������������������������������������������������������������������������������������������������������������������������������������������������������������������������������������������������������������������������������������������������������������������������������������������������������������������������������������������������������������������������������������������������������������������������������������������������������������������������������������������������������������������������������������������������������������������������������������������������������������������������������������������������������������������������������������������������������������������������������������������������������������������������������������������������������������������������������������������������������������������������������������������������������������������������������������������������������������������������������������������������������������������������������������������������������������������������������������������������������������������������������������������������������������������������������������������������������������������������������������������������������������

��������������������������������������������������������������������������������������������������������������������������������������������������������������������������������������������������������������������������������������������������������������������������������������������������������������������������������������������������������������

< Área da Seção do

Tubo

Área da Seção de

Escoamento

Área da Seção do

Tubo

• Nos textos de hidráulica é costume representar valores de vazão pelas letra Q ou q.

• Exemplo 3.1.1: Na figura abaixo são indicadas a velocidade média e a área no interior de três tubulações diferentes. Identifique a tubulação que transporta a maior vazão.

As vazões são: (a) 12Av, (b)14Av, e (c) 12Av. Desta forma, a maior vazão escoa através de b.

Page 3: 3. HIDRODINÂMICA - deg.ufla.brdeg.ufla.br/site/_adm/upload/file/hidrodinamicaPDF.pdf · 1 3. HIDRODINÂMICA 3.1 Princípios Gerais. • A Hidrodinâmica tem por objetivo geral o

3

• Exemplo 3.1.2: Verificou-se que a velocidade econômica de uma tubulação é 1,25m/s. Determine o diâmetro da tubulação para uma vazão é de 72m3/hora.

)Litros 1000m 1,0 que lembre ( s/L 20s

m02,0s3600

hh

m72Q 333

====

23

m016,0s/m25,1s/m02,0

VQ A VAQ ===⋅=

Em um tubo circular:

m1427,0m016,04A4D :logo e 4DA

22

=⋅π

⋅=

π⋅

=⋅π

=

• Exemplo 3.1.3: Considerando os diâmetros comerciais disponíveis no mercado, que são dados na tabela ao lado, selecione o tamanho comercial de tubo que resulta, para a vazão de 72m3/hora, na velocidade média de escoamento mais próxima de 1,25m/s. (lembre-se DN= diâmetro nominal, dem= diâmetro externo, e= espessura da parede).

De acordo com os dados da tabela, os valores de diâmetro interno e velocidade média de fluxo, para uma vazão de 72m3/hora ( 0.02m3/s), nos tubos de diâmetros nominais de 125, 150 e 200mm são:

125 mm⋅ 2 4.2× mm⋅− 116.6 mm= 4 0.020×m3

s⋅

π 0.1166 m⋅( )2×1.873

ms

=

150 mm⋅ 2 5× mm⋅− 140mm= 4 0.020×m3

s⋅

π 0.140 m⋅( )2×1.2992

ms

=

200 mm⋅ 2 5.4× mm⋅− 189.2 mm= 4 0.020×m3

s⋅

π 0.1892 m⋅( )2×0.7114

ms

=

Desta forma, a velocidade mais próxima de 1,25m/s é obtida com o tubo de DN=150.

LINHA FIXA LF SOLD. PN80NBR 14312

1,90 2,50 3,60 4,20 5,00 5,40

50,5 75,5

101,6 125,0 150,0 200,0

50 75

100 125 150 200

e (mm)dem (mm)DN

LINHA FIXA LF SOLD. PN80NBR 14312

1,90 2,50 3,60 4,20 5,00 5,40

50,5 75,5

101,6 125,0 150,0 200,0

50 75

100 125 150 200

e (mm)dem (mm)DN

Page 4: 3. HIDRODINÂMICA - deg.ufla.brdeg.ufla.br/site/_adm/upload/file/hidrodinamicaPDF.pdf · 1 3. HIDRODINÂMICA 3.1 Princípios Gerais. • A Hidrodinâmica tem por objetivo geral o

4

• Exemplo 3.1.4 : A figura abaixo indica a direção e o valor da vazão (em m3/h) escoando em quase todos os segmentos. Assumindo uma condição de fluxo permanente em todos os segmentos, determine a direção e o valor da vazão no segmento sem indicação.

Iniciando nocanto superior esquerdo no sentido horário, a vazão que entra no sistema é 5+ 4+8 + 4 =21 m3/h .Seguindo a mesma ordem, a vazão de saída é 2 + 6 =8 m3/h . Desta forma, se não existem outras perdas, a vazão no trecho sem identificação corre no sentido da saida e tem valor de 13 m3/h.

• Em um sistema com vazão constante, o princípio da conservação de massa resulta na seguinte igualdade:

332211 VAVAVA Q ⋅=⋅=⋅=

• Exemplo 3.1.5: Em uma pessoa normal em repouso, a area A0 da aorta na saída do coração tem um valor médio de 3 cm2 e a velocidade média do sangue v0 nesta seção e de 30 cm/s. Estime o número médio de capilares no corpo de uma pessoa considerando que cada capilar tem uma área média A de 3x10-7 cm2 e que no interior dos capilares o sangue escoa a uma velocidade v é de 0,05 cm/s.

A Linhas de corrente são linhas que são desenhadas no fluxo de forma

a auxiliar a visualizar a movimentação das partículas de fluído. As

3cm2 20⋅cms

60.0cm3

s=

n60

cm3

s⋅

3 10 7−⋅ cm2

⋅ 0.05⋅cms

:= n 4 109×=

Page 5: 3. HIDRODINÂMICA - deg.ufla.brdeg.ufla.br/site/_adm/upload/file/hidrodinamicaPDF.pdf · 1 3. HIDRODINÂMICA 3.1 Princípios Gerais. • A Hidrodinâmica tem por objetivo geral o

5

linhas de corrente são desenhadas de forma a serem tangentes a direção do vetor velocidade

Linha de corrente

Partícula de Fluido

Linha de corrente

Partícula de Fluido

• No fluxo permamente, a forma de cada linha de corrente não se altera ao longo do tempo .

Fluxo Permanente

Fluxo Variado

Fluxo Permanente

Fluxo Variado

t = 0 t = t+∆ t

Fluxo Permanente

Fluxo Variado

Fluxo Permanente

Fluxo Variado

t = 0 t = t+∆ t

• Em um dado instante, as linhas de corrente não podem cortar-se,

pois, em caso positivo, a partícula que se encontra no ponto de intersecção das linhas de corrente teria velocidades diferentes ao mesmo instante, o que não é possível.

Page 6: 3. HIDRODINÂMICA - deg.ufla.brdeg.ufla.br/site/_adm/upload/file/hidrodinamicaPDF.pdf · 1 3. HIDRODINÂMICA 3.1 Princípios Gerais. • A Hidrodinâmica tem por objetivo geral o

6

3.2 A Equação de Euler (1)

δtδs

• De uma maneira geral, quando uma partícula de fluído se desloca de A para B, as variação de velocidade (v) da particula são devido a mudanças no tempo (t) e na posição (s):

δtδvNo limite em que δt → 0, → dt

dv = a = aceleração

∂t∂v

dtdv = ∂v

∂s ·v +a = aceleração =

• No fluxo permanente, não existe variação da velocidade no tempo:

∂t∂v

dtdv = ∂v

∂s ·v +a = aceleração =

dtdv = ∂v

∂s ·v a = aceleração =

⇒δv = ∂t∂v·δs +∂v

∂s ·δt ∂t∂v+δt

δv = ∂v∂s · δt

δs

∂t∂v+dt

dv = ∂v∂s · δt

δsa = aceleração=

No limite em que δt→ 0, v = velocidade δtδsδtδs

• De uma maneira geral, quando uma partícula de fluído se desloca de A para B, as variação de velocidade (v) da particula são devido a mudanças no tempo (t) e na posição (s):

δtδvNo limite em que δt → 0, → dt

dv = a = aceleração

∂t∂v

dtdv = ∂v

∂s ·v +a = aceleração =

• No fluxo permanente, não existe variação da velocidade no tempo:

∂t∂v

dtdv = ∂v

∂s ·v +a = aceleração =

dtdv = ∂v

∂s ·v a = aceleração =

⇒δv = ∂t∂v·δs +∂v

∂s ·δt ∂t∂v+δt

δv = ∂v∂s · δt

δs

∂t∂v+dt

dv = ∂v∂s · δt

δsa = aceleração=

No limite em que δt→ 0, v = velocidade

• De uma maneira geral, quando uma partícula de fluído se desloca de A para B, as variação de velocidade (v) da particula são devido a mudanças no tempo (t) e na posição (s):

δtδvNo limite em que δt → 0, → dt

dv = a = aceleraçãoδtδvδtδvNo limite em que δt → 0, → dt

dv = a = aceleração→ dtdvdtdv = a = aceleração

∂t∂v

dtdv = ∂v

∂s ·v +a = aceleração = ∂t∂v∂t∂v

dtdv =dtdv = ∂v

∂s ·v +∂v∂s∂v∂s ·v +a = aceleração =

• No fluxo permanente, não existe variação da velocidade no tempo:

∂t∂v

dtdv = ∂v

∂s ·v +a = aceleração = ∂t∂v∂t∂v

dtdv =dtdv = ∂v

∂s ·v +∂v∂s∂v∂s ·v +a = aceleração =

dtdv = ∂v

∂s ·v a = aceleração = dtdv =dtdv = ∂v

∂s ·v ∂v∂s∂v∂s ·v a = aceleração =

⇒δv = ∂t∂v·δs +∂v

∂s ·δt ∂t∂v+δt

δv = ∂v∂s · δt

δs⇒δv = ∂t∂v·δs +∂v

∂s ·δt ∂t∂v+δt

δv = ∂v∂s · δt

δs⇒δv = ∂t∂v·δs +∂v

∂s ·δtδv = ∂t∂v∂t∂v·δs +∂v

∂s∂v∂s ·δt ∂t

∂v+δtδv = ∂v

∂s · δtδs

∂t∂v∂t∂v+δt

δv = ∂v∂s · δt

δs +δtδv =δtδv = ∂v

∂s · δtδs∂v

∂s∂v∂s · δt

δs· δtδsδtδs

∂t∂v+dt

dv = ∂v∂s · δt

δsa = aceleração=∂t∂v∂t∂v+dt

dv = ∂v∂s · δt

δs +dtdv =dtdv = ∂v

∂s · δtδs∂v

∂s∂v∂s · δt

δs· δtδsδtδsa = aceleração=

No limite em que δt→ 0, v = velocidade No limite em que δt→ 0, v = velocidade

Page 7: 3. HIDRODINÂMICA - deg.ufla.brdeg.ufla.br/site/_adm/upload/file/hidrodinamicaPDF.pdf · 1 3. HIDRODINÂMICA 3.1 Princípios Gerais. • A Hidrodinâmica tem por objetivo geral o

7

3.2 A Equação de Euler (2)

g·δm θ WS

g·δm·cos(θ)=Ws

δsδzg·δm· =Wscos(θ)= δz

δs

δm= δA ·δs·ρ

c) na direção s, a componente do peso Ws é dada por:

b) A razão entre a projeção de δs no eixo vertical (δ z) e δs é o cosseno de teta:

a) a massa deste volume elementar (δ m) é calculada pelo produto de seu volume pela sua massa específica (ρ) :

(p+δp)· δA

p·δA g·δm

a,v

δs

• Considere um volume elementar de água que se desloca na direção scom velocidade v e aceleração a.

• Se apenas as forças devido a pressão (p) e a gravidade (g) forem consideradas, o seguinte diagrama pode ser estabelecido:

θdireção s

z

Onde:p = pressãoA = áreag= aceleração da gravidade

δsδz =Ws

g·δA·δs·ρ·

++

θδsδz

g·δm θ WSg·δm θg·δm θ WS

g·δm·cos(θ)=Ws

δsδzg·δm· =Wsδsδzg·δm· =Wsδzg·δm· =Wscos(θ)= δz

δscos(θ)= δz

δs

δm= δA ·δs·ρ

c) na direção s, a componente do peso Ws é dada por:

b) A razão entre a projeção de δs no eixo vertical (δ z) e δs é o cosseno de teta:

a) a massa deste volume elementar (δ m) é calculada pelo produto de seu volume pela sua massa específica (ρ) :

(p+δp)· δA

p·δA g·δm

a,v

δs

• Considere um volume elementar de água que se desloca na direção scom velocidade v e aceleração a.

• Se apenas as forças devido a pressão (p) e a gravidade (g) forem consideradas, o seguinte diagrama pode ser estabelecido:

θdireção s

z θdireção s

z

Onde:p = pressãoA = áreag= aceleração da gravidade

δsδzδsδz =Ws

g·δA·δs·ρ·

++

θδsδz

θδsδz

Page 8: 3. HIDRODINÂMICA - deg.ufla.brdeg.ufla.br/site/_adm/upload/file/hidrodinamicaPDF.pdf · 1 3. HIDRODINÂMICA 3.1 Princípios Gerais. • A Hidrodinâmica tem por objetivo geral o

8

3.2 A Equação de Euler (3)

• Considerando agora o valor da aceleraçãono regime permamemte, a equação acima fica:

• Ao aplicar F=ma ( Newton) neste volume elementar, a reseltante na direção s é:

p·δA – (p+ ∂p)·δA - g·δA·δs·ρ· δsδz =δm·a

- ∂p·δA – g·δA·δs·ρ· δsδz = (δA·δs·ρ)·a

- ∂p – g·δs·ρ·δsδz= δs·ρ·a

• Dividindo a expressão acima pelo produto δs·ρ

δs∂p + g· + a = 0δs

∂zρ1 ·

dtdv = ∂v

∂s ·v a = aceleração =

+ g··δs∂p

+δs∂z

ρ1 ∂v

∂s ·v = 0

• Esta é a equação de Euler aplicada ao fluxo permanente. Lembre-se que na sua dedução só foram considerados os esforços devido a pressão e ao peso do fluido. As forças devido à viscosidade não foram consideradas.

(p+δp)· δA

p·δA g·δm

a,v

δs

δm= δA ·δs·ρ

θdireção s

z+

+

• Considerando agora o valor da aceleraçãono regime permamemte, a equação acima fica:

• Ao aplicar F=ma ( Newton) neste volume elementar, a reseltante na direção s é:

p·δA – (p+ ∂p)·δA - g·δA·δs·ρ· δsδz =δm·ap·δA – (p+ ∂p)·δA - g·δA·δs·ρ· δsδzp·δA – (p+ ∂p)·δA - g·δA·δs·ρ· δsδzδsδz =δm·a

- ∂p·δA – g·δA·δs·ρ· δsδz = (δA·δs·ρ)·a- ∂p·δA – g·δA·δs·ρ· δsδzδsδz = (δA·δs·ρ)·a

- ∂p – g·δs·ρ·δsδz= δs·ρ·a- ∂p – g·δs·ρ·δsδzδsδz= δs·ρ·a

• Dividindo a expressão acima pelo produto δs·ρ

δs∂p + g· + a = 0δs

∂zρ1 · δs

∂p + g· + a = 0δs∂z

ρ1 ·

dtdv = ∂v

∂s ·v a = aceleração = dtdv =dtdv = ∂v

∂s ·v ∂v∂s∂v∂s ·v a = aceleração =

+ g··δs∂p

+δs∂z

ρ1 ∂v

∂s ·v = 0+ g··δs∂p

+δs∂z

ρ1 ∂v

∂s ·v = 0

• Esta é a equação de Euler aplicada ao fluxo permanente. Lembre-se que na sua dedução só foram considerados os esforços devido a pressão e ao peso do fluido. As forças devido à viscosidade não foram consideradas.

(p+δp)· δA

p·δA g·δm

a,v

δs

δm= δA ·δs·ρ

θdireção s

z+

+

θdireção s

z θdireção s

z+

+

Page 9: 3. HIDRODINÂMICA - deg.ufla.brdeg.ufla.br/site/_adm/upload/file/hidrodinamicaPDF.pdf · 1 3. HIDRODINÂMICA 3.1 Princípios Gerais. • A Hidrodinâmica tem por objetivo geral o

9

3.3 A Equação de Bernoulli

• A equação de Euler pode ser integrada, reseultando na chamanda Equação de Bernoulli

i

0vsv

szg

sp1

=⋅∂∂

+∂∂

⋅+∂∂

⋅ρ

∫∫ ∫∫ =∂⋅+∂+∂ρ

ds0vvzgp1

tetanCons2

Vzgp1 2

=+⋅+⋅ρ

• Multiplicando todos os termos da equação acima por 1/g, obtemos a Equação de Bernoulli em termos de carga hidráulica (Energia por unidade de peso da água).

tetanConsg2

Vzggp

g1 2

=⋅

+⋅+⋅ρ⋅

especifico peso o representa' onde 1g

1 como γγ

=ρ⋅

tetanConsg2

VzP 2

=⋅

++γ

Page 10: 3. HIDRODINÂMICA - deg.ufla.brdeg.ufla.br/site/_adm/upload/file/hidrodinamicaPDF.pdf · 1 3. HIDRODINÂMICA 3.1 Princípios Gerais. • A Hidrodinâmica tem por objetivo geral o

10

• 3.4 Energia da Água e a Equação de Bernoulli

Energia Cinética (devido à velocidade)

Energia Potencial (devido ao posicionamento)

Na Hidrodinâmica consideramos que a energia total da água

tem três componentes. Energia de Pressão

(devido à pressão)

Energia = capacidade de realizar trabalho Trabalho = produto da força pelo deslocamento

unidade de trabalho =Newton metro = Joule

• Como a água escoa na forma de um corpo continuo, expressamos valores de energia da água em termos de energia por unidade de peso de água. Desta forma, podemos fazer um paralelo com os valores de energia utilizados no estudo da dinâmica de um corpo de massa “m”:

Componente da Energia Total Cálculo na Dinâmica

Cálculo em termos de Energia por unidade de

peso Potencial (Ep)

Enegia potencial de um copo de massa m

posicionado a um altura Z acima do referencial.

Z g mEp = Z g mZ g mEp ==

Cinética (Ec) Enegia Cinética de um

copo de massa m dotado de velocidade V.

2C Vm

21E ⋅⋅=

g2V

gm

Vm21

E2

2

C ⋅=

⋅⋅=

Pressão (Ep) No corpo de massa “m”

não é considerada γ

=PEp

Page 11: 3. HIDRODINÂMICA - deg.ufla.brdeg.ufla.br/site/_adm/upload/file/hidrodinamicaPDF.pdf · 1 3. HIDRODINÂMICA 3.1 Princípios Gerais. • A Hidrodinâmica tem por objetivo geral o

11

3.5 Teorema de Bernoulli para um Líquido Ideal (Conservação da Energia)

• No caso do líquido perfeito, ou ideal, não existe viscosidade, não havendo dissipação de energia durante o seu movimento. Na hipótese de não dissipação de energia, a soma das três componentes da energia é constante:

teconszg

VPzg

VP tan 2

2

2

222

1

211 =+

⋅+=+

⋅+

γγ

• Reprersentação gráfica •

������������������������������������������������

��������������������������������������������������

������������������������������������������������������������������������������������������������������������������������������������������������

������������������������������������������������

��������������������������������������������������������������������������������������������������������������������������

������������������������������������������������

������������������������������������������������

������������������������������������������������������������������������������������������������������������������������������������������������

������������������������������������������������

������������������������������������������������������������������������������������������������������������������������

������������������������

������������������������

����������������

•1 •3•2

Z1 Z2 Z3

gV

⋅2

21

gV

⋅2

22 g

V⋅2

23

γ1P

γ2P

γ3P

Linha de energiaPlano de carga efetivo

Linha Piezométrica

Plano de Referência

������������������������������������������������

��������������������������������������������������

������������������������������������������������������������������������������������������������������������������������������������������������

������������������������������������������������

��������������������������������������������������������������������������������������������������������������������������

������������������������������������������������

������������������������������������������������

������������������������������������������������������������������������������������������������������������������������������������������������

������������������������������������������������

������������������������������������������������������������������������������������������������������������������������

������������������������������������������������

��������������������������������������������������

������������������������������������������������������������������������������������������������������������������������������������������������

������������������������������������������������

��������������������������������������������������������������������������������������������������������������������������

������������������������������������������������

��������������������������������������������������

������������������������������������������������������������������������������������������������������������������������������������������������������������������������

��������������������������������������������������������������������������

������������������������������������������������������������������������������������������������������������������������������������������������

������������������������������������������������

��������������������������������������������������������������������������������������������������������������������������

������������������������������������������������

��������������������������������������������������������������������������������������������������������������������������

������������������������������������������������

������������������������������������������������

������������������������������������������������������������������������������������������������������������������������������������������������

������������������������������������������������

������������������������������������������������������������������������������������������������������������������������

������������������������������������������������

������������������������������������������������

������������������������������������������������������������������������������������������������������������������������������������������������������������������������

������������������������������������������������������������������������

������������������������������������������������������������������������������������������������������������������������������������������������

������������������������������������������������

������������������������������������������������������������������������������������������������������������������������

������������������������������������������������

������������������������������������������������������������������������������������������������������������������������

������������������������

������������������������

����������������

•1 •3•2

Z1 Z2 Z3

gV

⋅2

21

gV

⋅2

22 g

V⋅2

23

γ1P

γ2P

γ3P

Linha de energiaPlano de carga efetivo

Linha Piezométrica

Plano de Referência

• Exemplo de aplicação: velocidade de um jato de água.

Note que h=Z1-Z2

��������������������������������������������������������������������������������������������������������������������������������������������������������������������������������������������������������������������������������������������������������������������������������������������������������������������������������������������������������������������������������������������������������������������������������������������������������������������������������������������������������������������������������������������������������������������������������������������������������������������������������������������������������������������������������������������������������������������������������������������������������������������������������������������������������������������������������������������������������������������������������������������������������������������������������������������������������������������������������������������������������������������������������������������������������������������������������������������������������������������������������������������������������������������������������������������������������������������������������������������������������������������������������������������������������������������������������������������������������������������������������������������������������������������������������������������������������������������������������������������������������������������������������������������������������������������������������������������������������������������������������������������������������������������������������������������������������������������������������������������������������������������������������������������������������������������������������������������������������������������������������������������������������������������������������������������������������������������������������������������������������������������������������������������������������������������������������������������������������������������������������������������������������������������������������������������������������������������������������������������������������������������������������������������������������������������������������������������������������������������������������������������������������������������������������������������������������������������������������������������������������������������������������������������������������������������������������������������������������������������������������������������������������������������������������������������������������������������������������������������������������������������������������������������������������������������������������������������������������������������������������������������������������������������������������������������������������������������������������������������������������������������������������������������������������������������������������������������������������������������������������������������������������������������������������������������������������������������������������������������������������������������������������������������������������������������������������������������������������������������������������������������������������������������������������������������������������������������������������������������������������������������������������������������������������������������������������������������������������������������������������������������������������������������������������������������������������������������������������������������������������������������������������������������������������������������������������������������������������������������������������������������������������������������������������������������������������������������������������������������������������������������������������������������������������������������������������������������������������������������������������������������������������������������������������������������������������������������������������������������������������������������������������������������������������������������������������������������������������������������������������������������������������������������������������������������������������������������������������������������������������������������������������������������������������������������������������������������������������������������������������������������������������������������������������������������������������������������������������������������������������������������������������������������������������������������������������������������������������������������������������������������������������������������������������������������������������������������������������������������������������������������������������������������������������������������������������������������������������������������������������������������������������������������������������������������������������������������������������������������������������������������������������������������������������������������������������������������������������������������������������������������������������������������������������������������������������������������������������������������������������������������������������������������������������������������������������������������������������������������������������������������������������������������������������������������������������������������������������������������������������������������������������������������������������������������������������������������������������������������������������������������������������������������������������������������������������������������������������������������������������������������������������������������������������������������������������������������������������������������������������������������������������������������������������������������������������������������������������������������������������������������������������������������������������������������������������������������������������������������������������������������������������������������������������������������������������������������������������������������������������������������������������������������������������������������������������������������������������������������������������������������������������������������������������������������������������������������������������������������������������������������������������������������������������������������������������������������������������������������������������������������������������������������������������������������������������������������������������������������������������������

�����������������������������������������������������������������������������������������������������������������������������������������������������������������������������������������������������������������������������������������������������������������������������������������������������������������������������������������������������������������������������������������������������������������������������������������������������������������������

1

2Z1

Z2

γ1P

γ2P

gV

⋅2

22

hgV ⋅⋅= 22

Note que

• V1 é zero ou V12 <<< V2

2

•P1 = P2 ( pressão atmosférica)

2

2

22

21

2

222

11

gVzz

zg

VPzP

⋅=−

+⋅

+=+γγ

Plano de Referência

H1=H2

NA=cte

h

Note que h=Z1-Z2

��������������������������������������������������������������������������������������������������������������������������������������������������������������������������������������������������������������������������������������������������������������������������������������������������������������������������������������������������������������������������������������������������������������������������������������������������������������������������������������������������������������������������������������������������������������������������������������������������������������������������������������������������������������������������������������������������������������������������������������������������������������������������������������������������������������������������������������������������������������������������������������������������������������������������������������������������������������������������������������������������������������������������������������������������������������������������������������������������������������������������������������������������������������������������������������������������������������������������������������������������������������������������������������������������������������������������������������������������������������������������������������������������������������������������������������������������������������������������������������������������������������������������������������������������������������������������������������������������������������������������������������������������������������������������������������������������������������������������������������������������������������������������������������������������������������������������������������������������������������������������������������������������������������������������������������������������������������������������������������������������������������������������������������������������������������������������������������������������������������������������������������������������������������������������������������������������������������������������������������������������������������������������������������������������������������������������������������������������������������������������������������������������������������������������������������������������������������������������������������������������������������������������������������������������������������������������������������������������������������������������������������������������������������������������������������������������������������������������������������������������������������������������������������������������������������������������������������������������������������������������������������������������������������������������������������������������������������������������������������������������������������������������������������������������������������������������������������������������������������������������������������������������������������������������������������������������������������������������������������������������������������������������������������������������������������������������������������������������������������������������������������������������������������������������������������������������������������������������������������������������������������������������������������������������������������������������������������������������������������������������������������������������������������������������������������������������������������������������������������������������������������������������������������������������������������������������������������������������������������������������������������������������������������������������������������������������������������������������������������������������������������������������������������������������������������������������������������������������������������������������������������������������������������������������������������������������������������������������������������������������������������������������������������������������������������������������������������������������������������������������������������������������������������������������������������������������������������������������������������������������������������������������������������������������������������������������������������������������������������������������������������������������������������������������������������������������������������������������������������������������������������������������������������������������������������������������������������������������������������������������������������������������������������������������������������������������������������������������������������������������������������������������������������������������������������������������������������������������������������������������������������������������������������������������������������������������������������������������������������������������������������������������������������������������������������������������������������������������������������������������������������������������������������������������������������������������������������������������������������������������������������������������������������������������������������������������������������������������������������������������������������������������������������������������������������������������������������������������������������������������������������������������������������������������������������������������������������������������������������������������������������������������������������������������������������������������������������������������������������������������������������������������������������������������������������������������������������������������������������������������������������������������������������������������������������������������������������������������������������������������������������������������������������������������������������������������������������������������������������������������������������������������������������������������������������������������������������������������������������������������������������������������������������������������������������������������������������������������������������������������������������������������������������������������������������������������������������������������������������������������������������������������������������������������������������������������������������������������������������������������������������������������������������������������������������������������������������������������������������������������������������

�����������������������������������������������������������������������������������������������������������������������������������������������������������������������������������������������������������������������������������������������������������������������������������������������������������������������������������������������������������������������������������������������������������������������������������������������������������������������

1

2Z1

Z2

γ1P

γ2P

gV

⋅2

22

hgV ⋅⋅= 22

Note que

• V1 é zero ou V12 <<< V2

2

•P1 = P2 ( pressão atmosférica)

2

2

22

21

2

222

11

gVzz

zg

VPzP

⋅=−

+⋅

+=+γγ

Plano de Referência

H1=H2

NA=cte

h

Page 12: 3. HIDRODINÂMICA - deg.ufla.brdeg.ufla.br/site/_adm/upload/file/hidrodinamicaPDF.pdf · 1 3. HIDRODINÂMICA 3.1 Princípios Gerais. • A Hidrodinâmica tem por objetivo geral o

12

• Exemplo de Cálculo sob a hipótese de Líquido Ideal

Considerando que um líquido ideal escoa na tubulação esquematizada abaixo, calcule: (a) a velocidade da água imediatamente após passar pelo bocal localizado no final da tubulação (ponto 4); (b) a vazão (m3/s) ao longo da tubulação; (c) a altura ou carga de velocidade no ponto 2; (d) a altura ou carga de pressão no ponto 3

����������������������������������������������������������������������������������������������������������������������������������������������������������������������������������������������������������������������������������������������������������������������������������������������������������������������������������������������������������������������������������������������������������������������������������������������������������������������������������������������������

���������������������������������������������������������������������������������

������������������������������������������������������������������������

������������������������������������������������������������������������

��������������������������������������������������������������������������������������������������������������������������������������������������������������������������

��������������������������������������������������������

����������������������������������������

�������������������������������������������������������������������������

����������������������������������������������

������������������������������������������������������������������

������������������������������������������������������������������������������������������������������������������������������������������������������

�������������������������

����������������

Z4=5m

g2V2

4

⋅g

V⋅2

22 g

V⋅2

23

γ3P

Linha de energiaPlano de carga efetivo

Linha Piezométrica

Plano de Referência

γ2P

Z2=5m

A2= 36cm2 A3= 60cm2 A4= 6 cm2

2 Z3=5m

4

Z1=59m

Jato Livre

����������������

Z4=5m

g2V2

4

⋅g

V⋅2

22 g

V⋅2

23

γ3P

Linha de energiaPlano de carga efetivo

Linha Piezométrica

Plano de Referência

γ2P

Z2=5m

A2= 36cm2 A3= 60cm2 A4= 6 cm2

3

Z3=5mZ1=59m

Jato Livre

•1

Direção do fluxo

����������������������������������������������������������������������������������������������������������������������������������������������������������������������������������������������������������������������������������������������������������������������������������������������������������������������������������������������������������������������������������������������������������������������������������������������������������������������������������������������������

���������������������������������������������������������������������������������

������������������������������������������������������������������������

������������������������������������������������������������������������

��������������������������������������������������������������������������������������������������������������������������������������������������������������������������

��������������������������������������������������������

����������������������������������������

�������������������������������������������������������������������������

����������������������������������������������

������������������������������������������������������������������

������������������������������������������������������������������������������������������������������������������������������������������������������

�������������������������

���������������������������������������������������������������������������������

������������������������������������������������������������������������

������������������������������������������������������������������������

����������������������������������������������������������������������������������������������������������������������������������������������

������������������������������������������������������������������������

������������������������������������������������������������������������

��������������������������������������������������������������������������������������������������������������������������������������������������������������������������

��������������������������������������������������������

����������������������������������������

�������������������������������������������������������������������������

����������������������������������������������

������������������������������������������������������������������

������������������������������������������������������������������������������������������������������������������������������������������������������

�������������������������

����������������

Z4=5m

g2V2

4

⋅g2V2

4

⋅g

V⋅2

22

gV

⋅2

22 g

V⋅2

23

gV⋅2

23

γ3P

γ3P

Linha de energiaPlano de carga efetivo

Linha Piezométrica

Plano de Referência

γ2P

γ2P

Z2=5m

A2= 36cm2 A3= 60cm2 A4= 6 cm2

2 Z3=5m

4

Z1=59m

Jato Livre

����������������

Z4=5m

g2V2

4

⋅g2V2

4

⋅g

V⋅2

22

gV

⋅2

22 g

V⋅2

23

gV⋅2

23

γ3P

γ3P

Linha de energiaPlano de carga efetivo

Linha Piezométrica

Plano de Referência

γ2P

γ2P

Z2=5m

A2= 36cm2 A3= 60cm2 A4= 6 cm2

3

Z3=5mZ1=59m

Jato Livre

•1

Direção do fluxo

( )

( ) 53,46m0,54m-5m-59m P Zm59,81m/s2

m/s3,255P s/m255,3V

s/m55,32cm60

cm6 VVAVA Q :3 ponto no pressão de aargC )d(

m50,19,81m/s2

m/s5,425g2

V s/m425,5V

s/m55,32cm36

cm6 VVAVA Q :2 ponto no e velocidadde Carga (c)

/s0,0195m32,55m/s100006m Q VAVA Q : tubulação na Vazão)b(

s/m55,32 Vm)559(s/m81,92 V m5g2

V0m5900

0 Vtemos 1 em A4, que maior bem é io,reservatór de superfície a A1, como

zero) é relativa escala na aatmosféric pressão à referente valor(o 0PP como

Zg2

VPZg2

VP :energia da oconservaçã da Princípio

oconservaçã da princípio o com acordo de : 4 ponto no água da de Velocida)a(

312

23

3

2

2

34433

2

222

2

2

2

24422

32

4411

42

4

24

1

41

4

244

1

211

==γ

=+⋅

⋅+

γ=

⋅=⋅=⋅=

=⋅

⋅=

⋅=

⋅=⋅=⋅=

=⋅=⋅=⋅=

=−⋅⋅=+⋅

+=++

=

+⋅

=+⋅

Page 13: 3. HIDRODINÂMICA - deg.ufla.brdeg.ufla.br/site/_adm/upload/file/hidrodinamicaPDF.pdf · 1 3. HIDRODINÂMICA 3.1 Princípios Gerais. • A Hidrodinâmica tem por objetivo geral o

13

3.6 Teorema de Bernoulli para Líquidos Reais (Conservação da Energia) • No caso de um líquido real, que apresenta uma certa viscosidade,

existe dissipação de energia durante o movimento. Devido à dissipação da energia (perda de carga), a soma das três componentes da energia total não é constante:

212

222

1

211

2

2 −++

⋅+=+

⋅+ hfz

gVPz

gVP

γγ

onde hf 1-2 é a perda de carga entre os dois pontos considerados • Reprersentação gráfica:

������������������������������������������������������

Z2

gV

⋅2

21

gV

⋅2

22

γ1P

γ2P

Linha de energia

Plano de carga efetivo

Linha Piezométrica

���������������������������������������������������������������������������������������������������������������������������������������������������������������������������������������������������������������������������������������������������������������������������������������������������������������������������������������������������������������������������������������������������������������������������������������������������������������������������������������������������������������������

Z1

������������������������������

Direção do Fluxo:

Maior energia Menor Energia

hf1-2������������������������������������������������������

Z2

gV

⋅2

21

gV

⋅2

22

γ1P

γ2P

Linha de energia

Plano de carga efetivo

Linha Piezométrica

���������������������������������������������������������������������������������������������������������������������������������������������������������������������������������������������������������������������������������������������������������������������������������������������������������������������������������������������������������������������������������������������������������������������������������������������������������������������������������������������������������������������

Z1

������������������������������

Direção do Fluxo:

Maior energia Menor Energia

hf1-2

• Exemplo de aplicação: determinação da perda de carga admissível

em uma adutora por gravidade.

������������������������������������������������������������������������������������������������������������������������������������������������

Reservatório 1Z1

��������������������������������������������������������������������������������������������������������

Reservatório 2

γ3P

Linha de Energia Absolutaγ1P

Plano de carga absoluto

Z3

������������������������������������������������������������������������������������������������������������������������������������������������

������������������������������������������������������������������������������������������������������������������������������������������������

Reservatório 1Z1

��������������������������������������������������������������������������������������������������������

��������������������������������������������������������������������������������������������������������

Reservatório 2

γ3P

Linha de Energia Absolutaγ1P

Plano de carga absoluto

Z3

Page 14: 3. HIDRODINÂMICA - deg.ufla.brdeg.ufla.br/site/_adm/upload/file/hidrodinamicaPDF.pdf · 1 3. HIDRODINÂMICA 3.1 Princípios Gerais. • A Hidrodinâmica tem por objetivo geral o

14

3.7 Utilização do Teorema de Bernoulli em problemas envolvendo máquinas hidráulicas. Em problemas envolvendo bombas e turbinas, o teorema de Bernoulli, aplicado entre um ponto localizado na entrada da máquina (ponto E) e um ponto localizado na saida da máquina (ponto S), resulta em

Onde Hbomba representa a enegia que a bomba tranfere ao líquido que passa por ela. Ex: Hbomba=20 mca significa que cada Newton de água que passa através bomba tem a sua energia total aumentada de 20 Nm; Hturbina representa a enegia que o líquido transfere à tubina ao passar por ela. Ex: Hturbina=20 mca significa que cada Newton de água que passa através da turbina tem sua energia diimuida em 20 Nm; A aplicação da equação de Bernoulli na bomba esquematizada abaixo resulta em:

Bomba ZE

ZS

iBiA

Bomba ZE

iBiA

PE Ps

isiE

Bomba ZE

ZS

iBiA

Bomba ZE

iBiA

PE Ps

isiE

( ) ( ) SEEESSES

2E

2s

bomba

ssS

2s

SEbombaEEE

2E

hfiZiZppg2

VVH

zipg2

VhfHzipg2

V

++−++γ−

+−

=

+

+

γ+=−++

+

γ+

ss

2s

SEturbinabombaEE

2E zp

g2VhfHHzp

g2V

+=−−++γ

+ −

Page 15: 3. HIDRODINÂMICA - deg.ufla.brdeg.ufla.br/site/_adm/upload/file/hidrodinamicaPDF.pdf · 1 3. HIDRODINÂMICA 3.1 Princípios Gerais. • A Hidrodinâmica tem por objetivo geral o

15

Considerando que o trabalho efetuado por uma bomba é expresso pela da quantidade de energia (N x m = Joule) que deve ser transferida a cada unidade de peso (N) do líquido que por ela passa, a potência (trabalho por unidade de tempo N x m/s = Watt) é obtida pela seguinte expressão:

Onde PotH= Potência hidráulica da bomba (Watt=Nm/s); Hbomba= Energia total cedida pela bomba (mca= Nm / N de água); Q = vazão atrávés da bomba (m3/s); γ = Peso específico da água (N/m3). Obs: Hbomba, a energia cedida pela bomba por unidade de peso do líquido que por ela passa, é normalmente referida nos catálogos comerciais da bombas como HMT = altura manométrica total da bomba. Exemplo 3.7.1 : Calcule a potência hidráulica (em W) de uma bomba com vazão de 72m3/h de água (γ = 9806,65N/m3) e altura manométrica total de 50mca.

W65,9806m50mkgf65,9806

s3600h1

hm72Pot 3

3

H =⋅⋅⋅⋅

⋅⋅=

ou 9,807 kW Exemplo 3.7.2 : Calcule a potência hidráulica (em CV) de uma bomba com vazão de 72m3/h de água (γ =1000kgf/m3) e altura manométrica total de 50mca. Lembrando que 1 CV= 75kgf m/s

cv33,13s/mkgf75

cv1m50s

kgf1000s3600

h1h

m72Pot3

H =⋅⋅

⋅⋅⋅⋅⋅

⋅⋅

⋅=

Da mesma forma, utilizando o resultado anterior (exercício 3.7.1)e lembrando que 1kgf equivale a 9,80665N:

cv33,13s/mkgf75

cv1N80665,9

kgf1smN65,9806PotH =

⋅⋅⋅

⋅⋅

⋅⋅

⋅=

( )γ⋅⋅= QHPot bombaH

Page 16: 3. HIDRODINÂMICA - deg.ufla.brdeg.ufla.br/site/_adm/upload/file/hidrodinamicaPDF.pdf · 1 3. HIDRODINÂMICA 3.1 Princípios Gerais. • A Hidrodinâmica tem por objetivo geral o

16

Exemplo 3.7.3 : Calcule a potência hidráulica (em HP) de uma bomba com vazão de 72m3/h de água (γ =9806,65 N/m3) e altura manométrica total de 50mca. Lembrando que 1 HP= 550librasforça pés/s 1libra força (lbf) é o peso de uma massa de uma libra:

2** s/m80665,9kg45359237,0lbf ⋅⋅⋅= =4,44822....N 1pé = 12 polegadas = m3048,0m0254,012 * ⋅=⋅⋅

( ) s/péLbf550Hp1

m3048,0pé1

N80665,945359237,0Lbf

smN65,9806PotH ⋅⋅

⋅⋅⋅⋅

⋅⋅

=

HP151,13PotH ⋅=

Exemplo 3.7.4 : Calcule a energia cedida (Hbomba) e a potência hidráulica (em CV) da bomba esquematizada abaixo considerando um líquido ideal (hfA-B=0) de peso específico(γ) de 1000kgf/m3, com vazão (Q) de 400m3/h, leitura em um manômetro na entrada da bomba (PA) de 0,7kfg/cm2, leitura de um manômetro na saida da bomba (PB) de 3,0kgf/cm2e que o diâmetro da tubulação na entrada da bomba é de 300mm e que o diâmetro na saida da bomba é de 150mm.

Bomba ZE

ZS

iBiA

Bomba ZE

iBiA

PE Ps

isiE

Bomba ZE

ZS

iBiA

Bomba ZE

iBiA

PE Ps

isiE

Note que neste exercício :

( ) ( )

( ) ( )

γ−

+−

=

=+−+=

++−++γ−

+−

=

ES2E

2S

bomba

AEBS

SE

sEEESsES

2E

2S

bomba

ppg2VVH

nível) mesmo no estão manômetros (os 0iZiZideal) (Líquido 0hf

hfiZiZppg2VVH

Resposta = Hbomba =24,9 m e PotH= 36,9CV